Solve for p in the equation p − 12 = −27. 39 2 −39 −15

Answers

Answer 1

Answer:

P=-15

Step-by-step explanation:

P-12=-27  |+27

P+15=0

P=-15


Related Questions

Solve for the legs of the 45-45-90 triangle.

Answers

Answer:

1st answer option

Step-by-step explanation:

the 45° side angles make this an isoceles triangle (both legs are equally long).

therefore, x = y.

Pythagoras gives us

c² = a² + b²

with c being the Hypotenuse (the side opposite of the 90° angle), and a and b being the legs.

so, we have

(5 × sqrt(2))² = x² + y² (remember, x = y)

(5 × sqrt(2))² = x² + x²

25 × 2 = 2x²

25 = x²

x = 5

because of x = y

y = 5

If Triangle ABC has been translated 4 units to the right into Triangle DEF, what are the coordinates of Point E?

Answers

Triangle ABC has been translated 4 units to the right into triangle DEF. Then, the coordinates of point E is (6, -2).

What is the translation?

A translation in math moves a shape left or right and/or up or down. The translated shapes look exactly the same size as the original shape, and hence the shapes are congruent to each other. They just have been shifted in one or more directions.

Given that, triangle ABC has been translated 4 units to the right into Triangle DEF.

From the given coordinate plane A(-8, -2), B(2, -2) and (-4, -9)

When the shape is moved towards the right by k units, then replace x with x + k.

Now,

A(-8, -2) → D(-8+4, -2)=D(-4, -2)

B (2, -2) → E(2+4, -2)=E(6, -2)

C (-4, -9) → F(-4+4, -9)=F(0, -9)

Therefore, the coordinates of point E is (6, -2).

To learn more about the translation visit:

https://brainly.com/question/17485121.

#SPJ1

Which equation correctly solves for x in △RST?

Answers

The  equation that correctly solves for x in △RST is; x² + 11x - 42 = 0

How to find the lengths of similar triangles?

We are given that ΔDEF ∼ △RST.

This means that ΔDEF is similar to △RST.

From the concept of similar triangles, all corresponding sides are usually in the same ratio. Thus, applying this to our given triangles, we have;

DE/RS = EF/ST = DF/RT

This means when we plug in the relevant values, we will get;

(x + 5)/12 = (x + 7)/(x + 12) = 6/(x + 6)

Thus, we can say that;

(x + 5)/12 = 6/(x + 6)

Cross multiply to get;

(x + 5)(x + 6) = 72

x² + 11x + 30 - 72 = 0

x² + 11x - 42 = 0

Read more about similar triangles at; https://brainly.com/question/14285697

#SPJ1

Complete question is;

In the figure shown, ΔDEF ∼ △RST . Which equation can be used to find the value of x?

Each piece of lead for a mechanical pencil is 65 mm long. Find the total length in centimeters of the lead in a package with 25 pieces.

Answers

Answer:

cis theanswerforthequwstion

a club has 6 men and 2 women. a committee of two people is selected randomly. Find the expected number of women on the committee

Answers

The required expected number of women on the committee is 1/2 or 0.5.

The expected number of women on the committee can be found by calculating the probability that a woman is selected for the committee and then multiplying that probability by the number of people on the committee.

There are a total of 6 + 2 = 8 people in the club, and the committee has 2 people.

Given the committee is selected randomly, the probability that a particular person is selected for the committee is 2/8 = 1/4.

There are 2 women in the club, so the probability that a woman is selected for the committee is 2/8 = 1/4.

The expected number of women on the committee is the probability that a woman is selected for the committee multiplied by the number of people on the committee, which is (1/4) * 2 = 1/2.

Thus, the expected number of women on the committee is 1/2 or 0.5.

Learn more about probability here:

brainly.com/question/11234923

#SPJ1

find the price of a toy that wholesales for 2.25 and is marked up 100% for retail

Answers

100% of 2.25 is 2.25. It is marked up 100% so basically it’s just 2.25 times 2. 4.50 is your answer

toby buys a new mp3 player for the price of 45.5. what is the total amount his credit card is charged if the sales tax is 7%

Answers

45.50 x 1.07 = $48.69

Help answer the questions below
-----------------------------------------------------------------------------------------------------------
1. Reminder: Assume all dimensions are feet.
Task: Surround the given figure with a fence. Estimate that you will need 10% more than the actual perimeter in case of mistakes.
Cost: $26 for 4 feet of fence
Cost of Task =
----------------------------------------------------------------------------------------------------------
2. Reminder: Assume all dimensions are feet.
Task: Surround the given figure with a fence. Estimate that you will need 15% more than the actual perimeter in case of mistakes.
Cost: $25 for 4 feet of fence
Cost of Task =
-----------------------------------------------------------------------------------------------------------
3. Reminder: Assume all dimensions are feet.
Task: Surround the given figure with a fence. Estimate that you will need 20% more than the actual perimeter in case of mistakes.
Cost: $28 for 5 feet of fence
Cost of Task =
-----------------------------------------------------------------------------------------------------------
4. Reminder: Assume all dimensions are feet.
Task: Surround the given figure with a fence. Estimate that you will need 4% more than the actual perimeter in case of mistakes.
Cost: $27 for 4 feet of fence
Cost of Task =

Answers

The cost of the tasks to surround the figures with a fence, equivalent to   perimeter of the fence and an extra percentage are as follows;

1. $143

2. $143.75

3. $147.84

4. $224.64

What is the perimeter of a geometric figure?

The perimeter of a figure is the length of the line that surrounds the plane area covered by the figure.


1. The perimeter of the fence = 2 ft +  4 ft + 2 ft + 4 ft + 2 ft + 4 ft + (4 ft - 2 ft)  = 20 ft

The perimeter needed = 110/100 × 20 ft = 22 ft

The cost of 4 feet of fence = $26

Cost per feet of fence = $26/4 = $6.5/ft

Cost of the task = 22 feet × $6.5/ft = $143

2. The perimeter of the fence = 3 ft + 3 ft + 2 ft + 3 ft + 1 ft + 2 ft + 1 ft + 2 ft + 3 ft = 20 ft

The perimeter of the fence needed = 115/100 × 20 ft = 23 ft

Cost of 4 feet = $25

Cost per ft = $25/4 = $6.25

Cost of the fencing = 23 ft × $6.25/ft = $143.75

3. The perimeter of the fence = 5 ft +  3 ft + (6 ft - 5 ft) + 2 ft +  6 ft + 2 ft + 3 ft = 22 ft

The fence needed = 120/100 × 22 ft = 26.4 ft

The cost of 5 ft of fencing = $28

The cost per feet = $28/5 ft = $5.6/ft

The cost of the fencing = 26.4 ft × $5.6/ft = $147.84

Cost of the Task = $147.84

4. The perimeter of the fence = 8 ft + 6 ft + (8 ft - 6 ft) + (8 ft - 6 ft) + 6 ft + 8 ft = 32 ft

The fencing needed = 104/100 × 32 ft = 33.28 ft

The cost for 4 feet = $27

The cost per feet = $27/4 ft = $6.75/ft

The cost of the task = 33.28 ft × $6.75/ft = $224.64

Learn more about the perimeter of regular figures here:

https://brainly.com/question/29364161

#SPJ1

PLS LOOK AT PIC AND HELP ME!! I’m being timed pls help :((

Answers

A the y intercept will shift 7 up

Solve equation:
8n-7=-12+3n

Answers

Answer:

n = -1

Subtract and isolate

To solve this problem, the first thing we have to do is regroup the terms so that we can then subtract, simplify, subtract, and finally simplify again.

But first we must know.

¿What are the equations?

Equations are those mathematical equalities divided between two expressions which are called members and separated by their equal sign, in which known elements and unknown or unknown data appear, related by mathematical operations.

We solve the problem:8n - 7 = -12 + 3n8n - 7 = 3n - 128n - 7 - 3n = -125n - 7 = -125n = -12 + 75n = -5N = -1

Now we must check our results.

8n -7 = -12 + 3n

8 × - 1 - 7 = -12 + 3 × - 1

- 8 - 7 = - 12 - 3

- 15 = - 12 - 3

- 15 = - 15

So, our results are correct, the answer is n = - 1

¡Hope this helped!

What is the slope? Simplify your answer and write it as a proper fraction, improper fraction, or integer

Answers

Answer: 2/1

Step-by-step explanation:

You can decide this by looking at the rise over the run.

Solve equation
176 =-4(4 + 6m)

Answers

to solve for m

distribute into parentheses

176 = -16 - 24m

get m on one side

176 + 16 = -24m

192 = -24m

divide by -24

-8 = m

m = -8

Please help me on this I will give brainliest if possible

Answers

Answer:216

Step-by-step explanation:

50 POINTS ASAP!!!!!!!!!!!!!!!! 50 POINTS ASAP!!!!!50 POINTS ASAP!!!!!50 POINTS ASAP!!!!!!!!!!!!!!!! 50 POINTS ASAP!!!!!50 POINTS ASAP!!!!!

Which of the following statements about BC and BD is true?

Answers

The answer is A
Where BD has 8 units while
BC has 6 units and hence
BD is longer than BC

Answer: Its. C. Line BC is longer than line BD

Step-by-step explanation:

Find the measure of ∠ 1
a 180°
b 30°
c 60°
d 120°

Answers

Answer:

120

Step-by-step explanation:

Using the theorems of geometry, you know that all angles of a triangle add up to 180º. We can use an equation after looking at both angles, and the total:

30 + 30 + x = 180

60 + x = 180

x = 180 - 60

x = 120

40 POINTS ASAP!!!!!!!!!!!!!!!! 40 POINTS ASAP!!!!!40 POINTS ASAP!!!!!40 POINTS ASAP!!!!!!!!!!!!!!!! 40 POINTS ASAP!!!!!40 POINTS ASAP!!!!!

In the diagram, the measures of ∠2, ∠6, and ∠7 are 25°. The measure of ∠1 is 155°. Are lines c and d parallel?

Answers

Answer: Yes

Step-by-step explanation:

Parallel means that the lines are a constant distance away from each other and they never touch. As you can see lines C and D never touch.

The Lines D and C are Parallel since Parallel lines never touch

Hope this helps!

HELP HELP MEEEEE PLEASE Oliver is knitting a Scarf with 76 rows. He plans to knit 8 rows each week Part A Oliver drew this array to determine how long it will take for him to complete the scarf. What division sentence does the drawing show? O A 76÷8=9 OB 76÷8=9 R4 C. 76 ÷ 9 = 8 R2 D. 76 ÷ 9 = 8 R6 CHOOSE WORDS FROM THE DROP-DOWN MENUS TO EXPLAIN HOW LONG IT'LL TAKE FOR HOWEVER TO COMPLETE THE SCARF Oliver Will knit eight rows each week for A 4 B 8 C 9 weeks after that he will still have a 4 B 6 c9 Rows to knit​

Answers

Answer:

Answer: C. Oliver will knit eight rows each week for nine weeks and then he will still have nine rows to knit.

Step-by-step explanation:

Type the correct answer in the box. Spell all words correctly. Consider these four statements about a line that passes through two points on a plane. Statement A: The line is on a different plane from the points. Statement B: The line lies on only one plane. Statement C: The line is on the same plane as the points. Statement D: The line passes through only one plane. Only statement is true. Reset Next

Answers

Statement C is correct

The line lies on the same plane as the points.

What is a 3d Plane?

A 3-d or three-dimensional plane is called a hyperplane.

The three axes X-axis, Y-axis, and Z-axis are mutually perpendicular to each other.

We can represent a plane in vector form using the following equation. (p — p₀) . n = 0

Let the line is passing through two points on a given plane. Then,  this is the line joining the two points and so all the points lying on this line must also lie on the plane. This clearly tells that the line completely lies on the same plane as the points.

the correct option is Statement C. The line is on the same plane as the points

To refer more about Planes visit:

https://brainly.com/question/27927590

#SPJ1

Sine, Cosine, Tangent question below

Answers

Answer:

3 / 5

Step-by-step explanation:

TOA-CAH-SOH

TOA-CAH-SOH are 3 trigonometry ratios. They are:

TOA - tan(theta) = Opposite ÷ Adjacent

CAH - cos(theta) = Adjacent ÷ Hypotenuse

SOH - sin(theta) = Opposite ÷ Hypotenuse

Solution

We are finding Cos(A)

Cos(A) = Adjacent ÷ Hypotenuse

= AB ÷ AC

=

[tex] \frac{30}{50} \\ = \frac{3}{5} [/tex]

Therefore Cos(A) = 3 / 5.

kevin has 5 fish in his fish tank.Jasmine has 4 times as many fish as kevin has.How many fish does Jasmine have?

Answers

Answer:

20

Step-by-step explanation:

5 x 4 = 20

Darren had 20 minutes to do a three problem quiz he spent 10 1/4 minutes on question A and 5 4/5 minutes on question B how much time did he have left for question C

Answers

Adding the times on A and B gets you 16 minutes and 3 seconds. 20 minutes minus 16 minutes and 3 seconds will leave you with

3 minutes, 57 seconds

A truck driver is driving from Nome, Alaska, to Death Valley, California. Because he is traveling between locations with extreme temperatures, he needs to check the weather continuously to make sure the gas in his truck remains in liquid form. The gas he uses freezes at −40° F and evaporates at 140° F.

Part A: Write an inequality to represent the temperatures at which the gas in the truck will remain in liquid form. (2 points)

Part B: Describe the graph of the inequality completely from Part A. Use terms such as open/closed circles and shading directions. Explain what the solutions to the inequality represent. (4 points)

Part C: In January 1989, the temperature in Nome, Alaska, dropped to −49° F. Would the gas in the driver's truck have remained in liquid form so he could have driven on this day? Why or why not? (4 points)

Answers

(A)  The required inequality is,  -40°F  <  x  <  140°F.

(C) The gas has freezing point of -40°F, that means below that temperature the gas will freeze.

What is inequality?

In mathematics, inequalities describe the relationship between two values that are not equal. Equal means to be equal, not. The "not equal symbol (≠)" is typically used to indicate that two values are not equal. But different inequalities are used to compare the values to determine whether they are less than or greater than.

(A) Let the liquid range is from -40°F to 140°F.

Let x be the range.

So, the required inequality is,

-40°F  <  x  <  140°F

(B) The graph of inequality is attached below.

(C) Since the gas has freezing point of -40°F, that means below that temperature the gas will freeze.

(-49°F < 40°F)

Hence, the gas will freeze.

To know more about inequality, click on the link

https://brainly.com/question/24372553

#SPJ1

As a consumer starts paying his loan, the interest tends to decrease.
A. True
B. False

Answers

Answer:

A. true

Step-by-step explanation:

I believe you pay most of your interest when you first receive the loan.

For the regions A and B shown in the graph:

Part A: Discuss the limits of integration.
Part B: Set up an integral expression that represents the total area.
Part C: Calculate the total area.

Answers

The area for the shaded region is obtained as 2 units.

How to find the area between two curves?

In order to find the area, first find the intersection points of the two curves. Then, the area of the region can be found by taking the integration from the intersection point.

The curves in the graph in the form of equation are as follows,

y = x² + 3                     (1)

x = (y - 5)² - 2

=> y = √(x+ 2) + 5       (2)  

Part A :

The intersection points of the two curves are (-1.5, 5.5), (-1, 4) and (0.5, 3.5).

Thus, to calculate the area the limit is from x = -1.5 to x = 0.5.

Part B :

The expression of the integral is given as,

[tex]\int\limits^{-1}_{-1.5} {(x^{2} + 3 ) - (5 + \sqrt{x + 2} )} \, dx + \int\limits^{0.5}_{-1} {(5 + \sqrt{x + 2} ) -(x^{2} + 3 ) } \, dx[/tex]

Part C :

The area is found by evaluating the integral as follows,

[tex]\int\limits^{-1}_{-1.5} {(x^{2} + 3 ) - (5 + \sqrt{x + 2} )} \, dx + \int\limits^{0.5}_{-1} {(5 + \sqrt{x + 2} ) -(x^{2} + 3 ) } \, dx[/tex]

= [-1/3 + 3.375/3 + 3(-1 - (-1.5))] - [5(-1 - (-1.5)) + 2/3([tex]1^{3/2} - 0.5^{3/2}[/tex])] + [5(0.5 - (-1)) -(0.125/3 + 1/3 + 3(0.5 - (-1))]

= 2.291 - [2.5 + 2/3 × 0.647] + [5 × 1.5 - 4.8716]

= 2.291 - 2.927 + 2.6284

= 1.9924

Which is approximately 2.

Hence, the area between the curves is given as 2 units.

To know more about area between curves click on,

https://brainly.com/question/25546169

#SPJ1

In the data set shown below, what is the value of the quartiles?
{56, 58, 64, 68, 72, 75, 78}
PLEASE LOOK AT THE NUMBERS CAREFULLY
A. Q1 = 58; Q2 = 68; Q3 = 73.5
B. Q1 = 61; Q2 = 68; Q3 = 73.5
C. Q1 = 75; Q2 = 68; Q3 = 58
D. Q1 = 58; Q2 = 68; Q3 = 75

Answers

Answer:

To find the value of the quartiles, we need to first find the median of the data set. The median is the middle value of the data set, or the value that separates the lower half of the data from the upper half. In this data set, the median is 68, as it is the middle value when the data is arranged in ascending order.

Once we have the median, we can find the first and third quartiles. The first quartile, also known as Q1, is the median of the lower half of the data set, and the third quartile, also known as Q3, is the median of the upper half of the data set. In this data set, the lower half of the data is {56, 58, 64}, and the upper half of the data is {72, 75, 78}. The median of the lower half is 58, and the median of the upper half is 75.

Therefore, the value of the quartiles is Q1 = 58, Q2 = 68, and Q3 = 75.

Option D, Q1 = 58; Q2 = 68; Q3 = 75, is the correct answer.

Step-by-step explanation:

Use synthetic division to find the result when 2x4 - 8x³ - 3x² +28x - 3 is
divided by x-3.

Answers

you think I know

Step-by-step explanation:

bro what

Mary says the digits in the quotient of 381.109+0.86 are 44315.
but she doesn't know where to place the decimal point. How can
Mary use number sense to place the decimal point?

Answers

The correct place to the decimal is after 4 digits, that is 4431.5

What is decimal?

Decimals are the numbers, having two parts, a whole number part and a fractional part separated by a point that point is called the decimal. For example, 7.5 is a decimal number.

Given that, Mary says the digits in the quotient of 381.109÷0.86 are 44315.

but she doesn't know where to place the decimal point.

The division is 381.109/0.86

Placing the decimals in numerator and removing from denominator,

Since, in the denominator the decimal is two digits before, so multiply numerator and denominator by 100,

38110.9/86

According to rule, put the decimal point in the answer at the same place it is at in the dividend,

38110.9/86 = 4431.5

Hence, she should put decimal before digit 5.

For more references on decimals, click;

https://brainly.com/question/29765582

#SPJ1

An angle measures 76.6° less than
the measure of its
complementary
angle. What is the measure of each
angle?

Answers

the answer will be 45.6

Which ordered pair is a solution to the inequality 3x - 4y < 16?
(-3, -3)
(0, -4)
(4, -1)
(2, -3)

Answers

Answer: (-3,-3)

Step-by-step explanation:

Sub in the numbers

3(0)-4(-4)<16

16<16 wrong

3(-3)-4(-3)<16

-9+12<16

3<16 Correct

(-3,-3)

What’s the difference between fundamental theorem of calculus part 1 and 2? (Simple explanation pls bc I don’t understand)

Answers

The difference between fundamental theorem of calculus part 1 and 2 is explained below:

What is fundamental theorem of calculus?

The basic theorem of calculus is a theorem that connects the ideas of differentiating and integrating functions. A constant value that relies on where one begins to compute area makes the two procedures the inverses of one another. The relationship between the integral and derivative is established by the calculus fundamental theorem. Simply put, it says that the height of the area at a point x equals the rate of change of the area under the curve up to that point. We can find definite integrals with the aid of this theorem.

The connection between the integral and the derivative is demonstrated in The Fundamental Theorem of Calculus, Part 1. Take note. A definite integral can be assessed in terms of an antiderivative of its integrand using the Fundamental Theorem of Calculus, Part 2. This formula will calculate the total area under a curve.

To know more about Fundamental Theorem of Calculus, visit:

https://brainly.com/question/2293034

#SPJ1

Other Questions
Oral infections with herpes simplex virus 1 can lie dormant in nerve cells for years When a patient becomes stressed the virus is released, forming cold sores on the lips Why would the virus form a cold sore? Under the Equal Pay Act, an employer can legitimately pay different wages to male and female employees on the basis of a. the primary duties of the jobs. b. all of the choices. c. a seniority or merit system. d. any factor other than gender. Without graphing, find the slope of the line that goes through(0, 5) and (8, 2) daniel is part of a group and has a different opinion from the rest of the group. daniel can successfully influence the majority group through: PLEASE HELP!! USE C++ PLEASE!!Make a program that asks the user for an integer greater than zero (make sure that its greater than zero), then checks whether or not that number is a happy number (explanation below). For this program, you must use at least one recursive function to help determine whether or not the number is a happy number. To determine whether or not a number is a happy number, you take the sum of the squares of its digits. Then you take that sum, and you find the sum of the squares of its digits. You repeat this process until you end up at 1, which means its a happy number, or until you get stuck in a repeating cycle, which means its not a happy number. The repeating cycle is always 41637588914542204 To summarize, find the sum of the squares of the digits until the sum is equal to either 1 (happy number) or 4 (not happy number). The first 10 happy numbers are: 1, 7, 10, 13, 19, 23, 28, 31, 32, and 44 Example: 7349+9=5825+64=8964+81=1451+16+25=4216+4=204 73 makes it to 4, so 73 is not a happy number. 329+4=131+9=101+0=1 32 makes it to 1, so 32 is a happy number. 9. Gianna is taking a walking tour in her city. The entire tour is10 1/10 kilometers long. According to an app on her phone, Gianna's-average walking rate is 1.6 meters per second.A. How many meters does Gianna walk each hour?B. How many kilometers does Gianna walk each hour? According to the elaboration likelihood model, the two main determinants of a listeners willingness to engage in elaborating on the message are __________and _________. how to pay for social insurance programs administered by the social security administration, employers, employees, and the unemployed must contribute? (ii) Describe one potential negative consequence of this proposed project.BoldItalicUnderline Paying dividends to stockholders reduces taxable income because dividends are an expense.a. Trueb. False per osha, which responders are most likely to approach the point of a release in order to plug, patch, or otherwise stop the release? in research using surveillance camera footage of public places, researchers observed actual fighting behavior and found that third parties transactions that dont increase or decrease cash, but that result in significant investing and financing activities, are reported either directly after the cash flow statement or in a separate note to the financial statements as noncash activities. Please help me, this was on my sparx maths homework. It's due tomorrow. Margo was paid by companies to visit their stores and complete a questionnaire about the stores service quality. For instance, did the sales associate greet customers when they entered the store, and did they offer merchandise that complemented their purchases? Margo was T/FOrgans that receive both sympathetic and parasympathetic motor impulses are said to have dual innervation which is most likely to form a negative ion? none of the answers stated an element from group 17 an element from group 1 a metal Quadrilateral ABCD has coordinates A(2, 0), B(0, 4), C(4, 6), and D(2, 2).What are the coordinates of the image of ABCD after applying the transformation (x, y) (x, y) and then applying (x, y) (x, y)? How can a student like you benefit from performing cheer dance? the partners realize that brendan will be the first partner to start receiving cash. how much cash will brendan receive before any of the other partners collect any cash?